LSAT and Law School Admissions Forum

Get expert LSAT preparation and law school admissions advice from PowerScore Test Preparation.

User avatar
 Dave Killoran
PowerScore Staff
  • PowerScore Staff
  • Posts: 5849
  • Joined: Mar 25, 2011
|
#46251
Complete Question Explanation
(The complete setup for this game can be found here: lsat/viewtopic.php?t=14384)

The correct answer choice is (E)

If H is assigned to team 2, then from the final rule Q must be assigned to the middle position on team 1. This fact eliminates answer choices (A), (B), and (C). Answer choice (D) can be eliminated since from the first rule G cannot be assigned to the rear position on either team. Accordingly, answer choice (E) is correct.

Get the most out of your LSAT Prep Plus subscription.

Analyze and track your performance with our Testing and Analytics Package.